Which one of the following could be a complete and accurate list of the stations that stay open?

Bola on September 13 at 10:39AM

DEC 2017 SEC 4 GAME 3 SETUP

Is there a reason we have to set up rule 1 as R in -- N out? Since the game stated N&R cannot both close. I had my set up as N in R out and that is making have a different deduction when R is out. I am making a deduction that M is out when R is out. please explain why the set up says M is out when R is out.

Reply
Create a free account to read and take part in forum discussions.

Already have an account? log in

Emil-Kunkin on September 13 at 02:21PM

Those two potential articulations are contrapositives of each other, they mean the exact same thing! I would definitely suggest you take the contrapositive of conditional rules before making deductions, it often makes it easier to see how the rules fit together.
Let's look at what happens when R is out. If r is out then N is in, which means l is in too.